Next Article in Journal
Statistical Modeling Using a New Distribution with Application in Health Data
Next Article in Special Issue
Characterizing q-Bessel Functions of the First Kind with Their New Summation and Integral Representations
Previous Article in Journal
Eigenvalue Problem for a Reduced Dynamo Model in Thick Astrophysical Discs
Previous Article in Special Issue
Non-Emptiness, Relative Coincidences and Axiomatic Results for the Precore
 
 
Font Type:
Arial Georgia Verdana
Font Size:
Aa Aa Aa
Line Spacing:
Column Width:
Background:
Article

A Series Expansion of a Logarithmic Expression and a Decreasing Property of the Ratio of Two Logarithmic Expressions Containing Sine

1
School of Mathematics and Informatics, Henan Polytechnic University, Jiaozuo 454010, China
2
School of Information Science and Technology, Hainan Normal University, Haikou 571158, China
3
Independent Researcher, Dallas, TX 75252-8024, USA
*
Authors to whom correspondence should be addressed.
Mathematics 2023, 11(14), 3107; https://doi.org/10.3390/math11143107
Submission received: 14 June 2023 / Revised: 29 June 2023 / Accepted: 11 July 2023 / Published: 14 July 2023

Abstract

:
In the paper, by virtue of a derivative formula for the ratio of two differentiable functions and with the help of a monotonicity rule, the authors expand a logarithmic expression involving the sine function into the Maclaurin power series in terms of specific determinants and prove a decreasing property of the ratio of two logarithmic expressions containing the sine function. These results are interesting purely in pure mathematics.

1. Motivations

In [1] (pp. 42 and 55), we find the Maclaurin power series expansions
sin x = k = 0 ( 1 ) k x 2 k + 1 ( 2 k + 1 ) ! = x x 3 6 + x 5 120 x 7 5040 + x 9 362 , 880 , x R
and
ln sin x = ln x k = 1 2 2 k 1 ( 2 k ) ! k | B 2 k | x 2 k = ln x x 2 6 x 4 180 x 6 2835 , 0 < x < π ,
where B 2 k denotes the Bernoulli numbers, which can be generated in [2] (p. 3) by
z e z 1 = k = 0 B k z k k ! = 1 z 2 + k = 1 B 2 k z 2 k ( 2 k ) ! , | z | < 2 π .
For more information about B 2 k , please refer to the paper [3], and closely related references therein. From the series expansion (2), we acquire
ln sin x x = k = 1 2 2 k 1 ( 2 k ) ! k | B 2 k | x 2 k = x 2 6 x 4 180 x 6 2835 , 0 < | x | < π .
Motivated by the questions at the sites https://mathoverflow.net/q/444321 (accessed on 15 June 2023), https://mathoverflow.net/q/444490 (accessed on 12 April 2023), https://mathoverflow.net/q/444525 (accessed on 16 April 2023), and their answers therein, we consider the following two problems in this paper:
  • What is the Maclaurin power series expansion of the even function
    Q ( x ) = { ln 6 x 2 1 sin x x , 0 < | x | < 0 , x = 0
    around x = 0 ?
  • Is the even function
    R ( x ) = { ln 6 x 2 1 sin x x ln sin x x , | x | ( 0 , π ) 3 10 , x = 0 0 , x = ± π
    decreasing on the close interval [ 0 , π ] ?
It is clear that the logarithmic expression Q ( x ) defined in (3) is more complicated than the logarithmic expression ln sin x x . Therefore, it is interesting purely in pure mathematics to expand Q ( x ) into a Maclaurin power series expansion around the origin x = 0 and to compare these two logarithmic expressions by considering the decreasing property of their quotient R ( x ) .
In this paper, after preparing the general formula for higher derivatives of the quotient of two differentiable functions and reciting two monotonicity rules in Section 2, we will give an answer to the first problem by presenting a Maclaurin power series expansion of the even function Q ( x ) around x = 0 in Section 3, as well as provide an answer to the second problem by verifying the decreasing property of the quotient R ( x ) in Section 4.

2. Lemmas

For smoothly solving the above two problems, we need the following lemmas.
Lemma 1.
Let u ( x ) and v ( x ) 0 be two n-time differentiable functions on an interval I for a given integer n 0 . Then, the nth derivative of the ratio u ( x ) v ( x ) is
d n d x n u ( x ) v ( x ) = ( 1 ) n W ( n + 1 ) × ( n + 1 ) ( x ) v n + 1 ( x ) , n 0 ,
where the matrix
W ( n + 1 ) × ( n + 1 ) ( x ) = U ( n + 1 ) × 1 ( x ) V ( n + 1 ) × n ( x ) ( n + 1 ) × ( n + 1 ) ,
the matrix U ( n + 1 ) × 1 ( x ) is an ( n + 1 ) × 1 matrix whose elements satisfy u k , 1 ( x ) = u ( k 1 ) ( x ) for 1 k n + 1 , the matrix V ( n + 1 ) × n ( x ) is an ( n + 1 ) × n matrix whose elements are
v , j ( x ) = 1 j 1 v ( j ) ( x ) , j 0 0 , j < 0
for 1 n + 1 and 1 j n , and the notation | W ( n + 1 ) × ( n + 1 ) ( x ) | denotes the determinant of the ( n + 1 ) × ( n + 1 ) matrix W ( n + 1 ) × ( n + 1 ) ( x ) .
The formula (5) is a reformulation of [4] (p. 40, Exercise 5). See also the paper [5] and those papers collected at the wordpress.com site (https://qifeng618.wordpress.com/2020/03/22/some-papers-authored-by-dr-prof-feng-qi-and-utilizing-a-general-derivative-formula-for-the-ratio-of-two-differentiable-functions, accessed on 10 May 2023).
Lemma 2.
(Monotonicity rule for the ratio of two Maclaurin power series [6]). Let α k and β k for k { 0 } N be real numbers and the power series
U ( x ) = k = 0 α k x k and V ( x ) = k = 0 β k x k
be convergent on ( R , R ) for some R > 0 . If β k > 0 and the ratio α k β k is (strictly) increasing for k 0 , then the function U ( x ) V ( x ) is also (strictly) increasing on ( 0 , R ) .
Lemma 3.
(Monotonicity rule for the ratio of two functions [7] (Theorem 1.25)). For a , b R with a < b , let λ ( x ) and μ ( x ) be continuous on [ a , b ] , differentiable on ( a , b ) , and μ ( x ) 0 on ( a , b ) . If the ratio λ ( x ) μ ( x ) is increasing on ( a , b ) , then both λ ( x ) λ ( a ) μ ( x ) μ ( a ) and λ ( x ) λ ( b ) μ ( x ) μ ( b ) are increasing in x ( a , b ) .

3. Maclaurin Power Series Expansion

In this section, we solve the first problem posed in the first section of this paper.
Theorem 1.
Let the real numbers
e m = ( 1 ) m ( 2 m ) ! ( 2 m + 3 ) ! , m 0
and the determinants
E 2 n = A 2 n 1,1 B 2 n 1,2 n 1 e n + 1 e 0 C 1,2 n 1 , n 1 ,
where the matrices A 2 n 1 , 1 , B 2 n 1 , 2 n 1 , and C 1 , 2 n 1 for n 1 are defined by
A 2 n 1 , 1 = e 1 e 0 0 e 2 e 0 e n 1 e 0 0 e n e 0 = a i , j 1 i 2 n 1 j = 1 , a i , 1 = { e k e 0 , 1 i = 2 k 1 2 n 1 ; 0 , 2 i = 2 k 2 n 2 , B 2 n 1 , 2 n 1 = 1 1 0 0 0 0 0 2 2 0 0 0 3 1 e 1 e 0 0 0 0 0 2 n 3 1 e n 2 e 0 0 2 n 3 2 n 3 0 0 0 2 n 2 2 e n 2 e 0 0 2 n 2 2 n 2 0 2 n 1 1 e n 1 e 0 0 2 n 1 2 n 3 e 1 e 0 0 2 n 1 2 n 1 = b i , j 1 i , j 2 n 1 , b i , j = { 0 , 1 i < j 2 n 1 ; i j e k e 0 , 0 i j = 2 k 2 n 2 ; 0 , 1 i j = 2 k 1 2 n 1 , C 1 , 2 n 1 = 2 n + 1 1 e n e 0 0 2 n + 1 3 e n 1 e 0 2 n + 1 2 n 3 e 2 e 0 0 2 n + 1 2 n 1 e 1 e 0 = c i , j i = 1 1 j 2 n 1 , c 1 , j = { 2 n + 1 2 k 1 e n k + 1 e 0 , 1 j = 2 k 1 2 n 3 ; 0 , 2 j = 2 k 2 n 2 .
Then the function Q ( x ) defined by (3) can be expanded into the Maclaurin power series expansion
Q ( x ) = 1 20 x 2 n = 1 E 2 n x 2 n + 2 ( 2 n + 2 ) ! = 1 20 x 2 1 16,800 x 4 + 1 756,000 x 6 + 89 3,104,640,000 x 8 +
for x R .
Proof. 
The first derivative of Q ( x ) is
Q ( x ) = 2 x 3 sin x + x cos x x ( sin x x ) = 3 sin   x 2 x x cos   x x 4 x sin   x x 3 ,
where
x sin x x 3 = k = 0 e k x 2 k ( 2 k ) ! , 0 < | x | <
and
3 sin x 2 x x cos x x 4 = k = 0 e k + 1 x 2 k + 1 ( 2 k + 1 ) ! , 0 < | x | < .
Hence, the functions
v ( x ) = { x sin x x 3 , x 0 e 0 , x = 0 and u ( x ) = { 3 sin x 2 x x cos x x 4 , x 0 0 , x = 0
satisfy
v ( k ) ( 0 ) = { e m , k = 2 m 0 , k = 2 m + 1 and u ( k ) ( 0 ) = { 0 , k = 2 m e m + 1 , k = 2 m + 1
for k , m 0 . Accordingly, making use of Formula (5) results in
Q ( 2 n + 2 ) ( 0 ) = lim x 0 u ( x ) v ( x ) ( 2 n + 1 ) = ( 1 ) 2 n + 1 v 2 n + 2 ( 0 ) u ( 0 ) v ( 0 ) 0 0 u ( 0 ) v ( 0 ) 1 1 v ( 0 ) 0 u ( 0 ) v ( 0 ) 2 1 v ( 0 ) 0 u ( 3 ) ( 0 ) v ( 3 ) ( 0 ) 3 1 v ( 0 ) 0 u ( 2 n 2 ) ( 0 ) v ( 2 n 2 ) ( 0 ) 2 n 2 1 v ( 2 n 3 ) ( 0 ) 0 u ( 2 n 1 ) ( 0 ) v ( 2 n 1 ) ( 0 ) 2 n 1 1 v ( 2 n 2 ) ( 0 ) 0 u ( 2 n ) ( 0 ) v ( 2 n ) ( 0 ) 2 n 1 v ( 2 n 1 ) ( 0 ) 2 n 2 n v ( 0 ) u ( 2 n + 1 ) ( 0 ) v ( 2 n + 1 ) ( 0 ) 2 n + 1 1 v ( 2 n ) ( 0 ) 2 n + 1 2 n v ( 0 ) = 1 e 0 2 n + 2 0 e 0 0 0 0 e 1 0 1 1 e 0 0 0 0 e 1 0 2 2 e 0 0 e 2 0 3 1 e 1 0 0 0 e n 1 0 2 n 2 2 e n 2 0 e n 0 2 n 1 1 e n 1 0 0 0 e n 0 2 n 2 e n 2 n 2 n e 0 e n + 1 0 2 n + 1 1 e n 0 0 = 1 e 0 2 n + 1 e 1 1 1 e 0 0 0 0 0 0 0 2 2 e 0 0 0 0 e 2 3 1 e 1 0 0 0 0 0 0 2 n 2 2 e n 2 2 n 2 2 n 2 e 0 0 0 e n 2 n 1 1 e n 1 0 0 2 n 1 2 n 1 e 0 0 0 0 2 n 2 e n 1 2 n 2 n 2 e 1 0 2 n 2 n e 0 e n + 1 2 n + 1 1 e n 0 0 2 n + 1 2 n 1 e 1 0 = e 1 e 0 1 1 0 0 0 0 0 0 2 2 0 0 0 e 2 e 0 3 1 e 1 e 0 0 0 0 0 0 0 2 n 2 2 e n 2 e 0 2 n 2 2 n 2 0 0 e n e 0 2 n 1 1 e n 1 e 0 0 0 2 n 1 2 n 1 0 0 0 2 n 2 e n 1 e 0 2 n 2 n 2 e 1 e 0 0 2 n 2 n e n + 1 e 0 2 n + 1 1 e n e 0 0 0 2 n + 1 2 n 1 e 1 e 0 0 = e 1 e 0 1 1 0 0 0 0 0 0 2 2 0 0 0 e 2 e 0 3 1 e 1 e 0 0 0 0 0 e n 1 e 0 2 n 3 1 e n 2 e 0 0 2 n 3 2 n 3 0 0 0 0 2 n 2 2 e n 2 e 0 0 2 n 2 2 n 2 0 e n e 0 2 n 1 1 e n 1 e 0 0 2 n 1 2 n 3 e 1 e 0 0 2 n 1 2 n 1 e n + 1 e 0 2 n + 1 1 e n e 0 0 2 n + 1 2 n 3 e 2 e 0 0 2 n + 1 2 n 1 e 1 e 0 = A 2 n 1 , 1 B 2 n 1 , 2 n 1 e n + 1 e 0 C 1 , 2 n 1 = E 2 n
for n 1 and, due to the evenness of Q ( x ) on R , Q ( 2 n + 1 ) ( 0 ) = 0 for n 0 . Consequently, we obtain
Q ( x ) = n = 0 Q ( n ) ( 0 ) n ! x n = n = 1 Q ( 2 n ) ( 0 ) ( 2 n ) ! x 2 n = Q ( 0 ) 2 ! x 2 + n = 1 Q ( 2 n + 2 ) ( 0 ) ( 2 n + 2 ) ! x 2 n + 2 = 1 20 x 2 n = 1 E 2 n ( 0 ) ( 2 n + 2 ) ! x 2 n + 2 .
The required proof is thus complete. □
Remark 1.
For n = 3 , the determinant E 6 is
E 6 = e 1 e 0 1 1 0 0 0 0 0 0 2 2 0 0 0 e 2 e 0 3 1 e 1 e 0 0 3 3 0 0 0 0 4 2 e 1 e 0 0 4 4 0 e 3 e 0 5 1 e 2 e 0 0 5 3 e 1 e 0 0 5 5 e 4 e 0 7 1 e 3 e 0 0 7 3 e 2 e 0 0 7 5 e 1 e 0 = 1 10 1 0 0 0 0 0 0 1 0 0 0 1 35 3 10 0 1 0 0 0 0 3 5 0 1 0 1 84 1 7 0 1 0 1 1 165 1 12 0 1 0 21 10 = 89 77,000
and E 6 8 ! = 89 3 , 104 , 640 , 000 . This coincides with the coefficient of the term x 8 in the Maclaurin power series expansion (6).

4. Decreasing Property

In this section, we solve the second problem posed in the first section of this paper.
Theorem 2.
The function R ( x ) defined by (4) decreasingly maps [ 0 , π ] onto 0 , 3 10 .
First proof. 
Directly differentiating gives
Q ( x ) ln sin x x = ( 2 x 3 sin x + x cos x ) sin x ( sin x x ) ( x cos x sin x )
and
Q ( x ) ln sin x x = h ( x ) ( x sin x ) 2 ( x cos x sin x ) 2 .
where
h ( x ) = 2 x 3 + x 3 cos 3 x 4 x 2 sin 3 x 2 x 2 sin x cos x sin 3 x + 2 x sin 2 x 2 cos x sin 3 x + 7 x cos x sin 2 x 7 x 2 sin x cos 2 x .
In [1] (p. 43), we find
sin 2 x = k = 1 ( 1 ) k + 1 2 2 k 1 ( 2 k ) ! x 2 k , | x | < ,
sin 3 x = 1 4 k = 1 ( 1 ) k + 1 3 2 k + 1 3 ( 2 k + 1 ) ! x 2 k + 1 , | x | < ,
cos 3 x = 1 4 k = 0 ( 1 ) k 3 2 k + 3 ( 2 k ) ! x 2 k , | x | < .
Differentiating results in
sin x cos x = 1 2 k = 0 ( 1 ) k 2 2 k + 1 ( 2 k + 1 ) ! x 2 k + 1 , | x | < ,
cos x sin 2 x = 1 12 k = 1 ( 1 ) k + 1 3 2 k + 1 3 ( 2 k ) ! x 2 k , | x | < ,
and
sin x cos 2 x = 1 12 k = 0 ( 1 ) k 3 2 k + 2 + 3 ( 2 k + 1 ) ! x 2 k + 1 , | x | < .
Theorem 2.1 in the paper [8] reads that
sin z z = 1 + j = 1 ( 1 ) j T ( + 2 j , ) + 2 j ( 2 z ) 2 j ( 2 j ) !
for 0 and z C , where
T ( n , ) = 1 ! m = 0 ( 1 ) m m 2 m n .
Taking n = 4 + 2 j and = 4 in (15) gives
T ( 4 + 2 j , 4 ) = 1 4 ! m = 0 4 ( 1 ) m 4 m ( 2 m ) 4 + 2 j = 4 j + 1 1 3 .
Setting = 4 in (14) leads to
sin z z 4 = 1 + j = 1 ( 1 ) j T ( 4 + 2 j , 4 ) 4 + 2 j 4 ( 2 z ) 2 j ( 2 j ) ! = j = 0 ( 1 ) j 2 2 j + 3 ( 4 j + 1 1 ) ( 2 j + 4 ) ! z 2 j ,
which can be rearranged as
sin 4 z = j = 0 ( 1 ) j 2 2 j + 3 ( 2 2 j + 2 1 ) ( 2 j + 4 ) ! z 2 j + 4 , | z | < .
Differentiating gives
cos z sin 3 z = j = 0 ( 1 ) j 2 2 j + 1 ( 2 2 j + 2 1 ) ( 2 j + 3 ) ! z 2 j + 3 , | z | < .
Making use of the power series expansions (8)–(13) and (16), we can expand the function h ( x ) into
h ( x ) = 2 x 3 + 1 4 k = 0 ( 1 ) k 3 2 k + 3 ( 2 k ) ! x 2 k + 3 + k = 1 ( 1 ) k 3 2 k + 1 3 ( 2 k + 1 ) ! x 2 k + 3 k = 0 ( 1 ) k 2 2 k + 1 ( 2 k + 1 ) ! x 2 k + 3 1 4 k = 0 ( 1 ) k 3 2 k + 3 3 ( 2 k + 3 ) ! x 2 k + 3 + 2 k = 0 ( 1 ) k 2 2 k + 1 ( 2 k + 2 ) ! x 2 k + 3 2 k = 0 ( 1 ) k 2 2 k + 1 ( 2 2 k + 2 1 ) ( 2 k + 3 ) ! x 2 k + 3 + 7 12 k = 0 ( 1 ) k 3 2 k + 3 3 ( 2 k + 2 ) ! x 2 k + 3 7 12 k = 0 ( 1 ) k 3 2 k + 2 + 3 ( 2 k + 1 ) ! x 2 k + 3 = 1 2 k = 5 ( 1 ) k 4 k 3 6 k 2 + 29 k + 57 3 2 k 2 k 2 + 3 k 1 2 2 k + 3 2 4 k + 5 + 12 k 3 2 k 2 69 k 57 ( 2 k + 3 ) ! x 2 k + 3 = x 13 2 k = 0 ( 1 ) k + 1 4 k 3 + 54 k 2 + 269 k + 552 3 2 k + 10 2 k 2 + 23 k + 64 2 2 k + 13 2 4 k + 25 + 12 k 3 + 178 k 2 + 811 k + 1048 ( 2 k + 13 ) ! x 2 k x 13 2 k = 0 ( 1 ) k Θ k x 2 k .
We now consider the functions
Φ k ( x ) = ( 1 ) k Θ k x 2 k + ( 1 ) k + 1 Θ k + 1 x 2 k + 2 = ( 1 ) k x 2 k Θ k Θ k + 1 x 2
for k 0 and x [ 0 , π ] . Then the function h ( x ) can be expressed as
h ( x ) = x 13 2 k = 0 Φ 2 k ( x ) = x 13 2 k = 0 Θ 2 k + 1 Θ 2 k Θ 2 k + 1 x 2 x 4 k , x [ 0 , π ] .
By induction, we can verify that
2 4 k + 25 4 k 3 + 54 k 2 + 269 k + 552 3 2 k + 10 = 32 × 3 2 k + 10 4 3 2 k + 10 4 k 3 + 54 k 2 + 269 k + 552 32 > 0
and
2 k 2 + 23 k + 64 2 2 k + 13 ( 12 k 3 + 178 k 2 + 811 k + 1048 ) = 2 k 2 + 23 k + 64 2 2 k + 13 12 k 3 + 178 k 2 + 811 k + 1048 2 k 2 + 23 k + 64 > 0
for k 0 . Therefore, we acquire that Θ k > 0 for k 0 .
The partial sum
P ( x ) = k = 0 2 Θ 2 k + 1 Θ 2 k Θ 2 k + 1 x 2 x 4 k = 1 4200 x 2 18,000 + 361 x 4 58,212,000 19 x 6 43,243,200 + 338,857 x 8 15,256,200,960,000 1,331,567 x 10 1,556,132,497,920,000
satisfies
P ( x ) = 1,331,567 x 4 311,226,499,584,000 + 338,857 x 3 3,814,050,240,000 19 x 2 14,414,400 + 361 x 29,106,000 1 18,000 1 18,000 + 361 π 29,106,000 19 π 2 14,414,400 + 33 8,857 π 3 3,814,050,240,00 0 1 , 331,567 π 4 3 11,226,499,584,000 , x π < 0 , P ( x ) = 1,331,567 x 3 77,806,624,896,0 00 + 338 , 857 x 2 1,271,350,080,000 19 x 7,207,200 + 361 29,106,000 361 29,106,000 19 π 7,207,200 + 338,857 π 2 1 , 271,350,080 , 000 1,331,567 π 3 7 7,806,624,896,00 0 , x π > 0 , P ( x ) = 1 , 331,567 x 2 25,935,541,632,000 + 338,857 x 63 5,675,040,000 19 7,207,20 0 19 7,207,20 0 + 3 38,85 7 π 635,675,040,0 00 1,331,56 7 π 2 25,935,541,632,00 0 , x π < 0 , P ( 4 ) ( x ) = 338,857 6 35,675,040,0 00 1,331,56 7 x 12,967,770,816,000 3 38,857 635,675,040,000 1,331,567 π 12,967,770,816,000 , x π > 0 .
Accordingly, the function P ( x ) is decreasing on [ 0 , π ] . From
P ( π ) = 1 4200 π 18,000 + 361 π 2 58,212,000 19 π 3 43,243,200 + 338,857 π 4 15,256,200,960,00 0 1,331,567 π 5 1,556,132,497,920,000 = 0.000113 ,
it follows that P ( x ) > 0 on [ 0 , π ] .
The inequality
Θ 2 k Θ 2 k + 1 > 10 > π 2 , k 1
is equivalent to
Θ 2 k > 10 Θ 2 k + 1 , k 1 .
In other words, this inequality can be formulated as
8 k 2 + 58 k + 25 2 8 k + 25 ( 256 k 5 + 3584 k 4 + 18 , 752 k 3 + 46 , 420 k 2 + 53 , 469 k + 18 , 405 ) 3 4 k + 10 + 32 k 4 + 416 k 3 + 1930 k 2 + 3731 k + 2470 2 4 k + 14 768 k 5 11 , 264 k 4 63 , 872 k 3 172 , 940 k 2 219 , 064 k 99 , 795 = 32 8 k 2 + 58 k + 25 3 4 k + 10 [ 4 3 4 k + 10 256 k 5 + 3584 k 4 + 18,752 k 3 + 46,420 k 2 + 53,469 k + 18,405 32 ( 8 k 2 + 58 k + 25 ) ] + 32 k 4 + 416 k 3 + 1930 k 2 + 3731 k + 2470 × 4 2 k + 7 768 k 5 + 11 , 264 k 4 + 63 , 872 k 3 + 172 , 940 k 2 + 219 , 064 k + 99 , 795 32 k 4 + 416 k 3 + 1930 k 2 + 3731 k + 2470 > 0
for k 1 . By induction, we reveal that
4 3 4 k + 10 256 k 5 + 3584 k 4 + 18 , 752 k 3 + 46 , 420 k 2 + 53 , 469 k + 18 , 40 5 32 ( 8 k 2 + 58 k + 25 ) > 0
and
4 2 k + 7 768 k 5 + 11 , 264 k 4 + 63 , 872 k 3 + 172 , 940 k 2 + 219 , 064 k + 99 , 795 32 k 4 + 416 k 3 + 1930 k 2 + 3731 k + 2470 > 0
for k 1 . Consequently, the inequality (17) holds for k 1 .
Basing on the above-derived results, we conclude that
h ( x ) = x 13 2 P ( x ) + k = 3 Θ 2 k + 1 Θ 2 k Θ 2 k + 1 x 2 x 4 k > 0 , x [ 0 , π ] .
From (7), it follows that the derivative ratio, Q ( x ) ( ln sin x x ) is decreasing on ( 0 , π ) . Using Lemma 3, we derive that the function R ( x ) defined by (4) is decreasing on [ 0 , π ] . The proof of Theorem 2 is complete. □
Second proof. 
We just state the outline and sketch of this proof because the method and approach are similar to the first proof in this paper.
Differentiating gives
h ( x ) = 1 2 ( 15 x cos 5 x 2 3 x 3 cos 5 x 2 + 15 x cos 3 x 2 3 x 3 cos 3 x 2 30 x cos x 2 6 x 3 cos x 2 + 24 x 2 sin x 2 + 4 sin x 2 + 8 sin 3 x 2 + 14 x 2 sin 3 x 2 + 6 x 2 sin 5 x 2 4 sin 7 x 2 ) sin x 2 = x 11 sin x 2 150 × 2 2 k k = 5 ( 1 ) k + 1 150 × 7 2 k + 1 25 32 k 3 112 k 2 + 26 k + 81 3 2 k 9 ( 2 k + 1 ) 16 k 2 48 k + 125 5 2 k 300 48 k 3 48 k 2 51 k 7 ( 2 k + 1 ) ! x 2 ( k 5 ) x 11 sin x 2 2 2 k + 1 k = 0 ( 1 ) k W k x 2 k = x 11 sin x 2 2 2 k + 1 k = 0 W 2 k + 1 W 2 k W 2 k + 1 x 2 x 4 k
for x [ 0 , π ] where
W k = 2 × 7 2 k + 11 3 ( 2 k + 11 ) 16 k 2 + 112 k + 285 5 2 k + 8 32 k 3 + 368 k 2 + 1306 k + 1411 3 2 k + 9 4 48 k 3 + 672 k 2 + 3069 k + 4538 ( 2 k + 11 ) ! , k 0 .
By induction, we can verify that
3 2 k + 9 4 48 k 3 + 672 k 2 + 3069 k + 4538 32 k 3 + 368 k 2 + 1306 k + 1411 > 0 , k 0 , 7 2 k + 11 2 32 k 3 + 368 k 2 + 1306 k + 1411 3 2 k + 9 = 49 × 3 2 k + 9 7 3 2 k + 9 2 32 k 3 + 368 k 2 + 1306 k + 1411 49 > 0 , k 0 ,
and
7 2 k + 11 3 ( 2 k + 11 ) 16 k 2 + 112 k + 285 5 2 k + 8 = 343 × 5 2 k + 8 7 5 2 k + 8 3 ( 2 k + 11 ) 16 k 2 + 112 k + 285 343 > 0 , k 4 .
Moreover, it is easy to calculate
W 0 = 3328 7 , W 1 = 9088 21 , W 2 = 1 , 479 , 584 8085 , W 3 = 5 , 030 , 768 105 , 105 .
Therefore, the sequence W k is positive for k 0 .
The partial sum
k = 0 9 W 2 k + 1 W 2 k W 2 k + 1 x 2 x 4 k > 0 , x [ 0 , π ] .
For k 3 , the inequality
W 2 k W 2 k + 1 > 10 > π 2
is valid. Hence, the derivative h ( x ) is positive on [ 0 , π ] . Then the function h ( x ) is increasing on [ 0 , π ] . Since h ( 0 ) = 0 , the function h ( x ) is positive on ( 0 , π ] . From the derivative in (7), it follows that the derivative ratio Q ( x ) ( ln sin x x ) is decreasing on ( 0 , π ) . Employing Lemma 3, we deduce that the function R ( x ) defined by (4) is decreasing on [ 0 , π ] . The second proof of Theorem 2 is complete. □
Remark 2.
Directly differentiating, carefully rearranging, deliberately expanding, and making use of the series expansion (1) give
Q ( x ) ln sin x x = ( 2 x 3 sin x + x cos x ) sin x ( sin x x ) ( x cos x sin x ) = 2 x sin x + x 2 sin ( 2 x ) + 3 2 cos ( 2 x ) 3 2 x sin x + x 2 sin ( 2 x ) + 1 2 cos ( 2 x ) x 2 cos x 1 2 = k = 3 ( 1 ) k + 1 ( k 3 ) 2 2 k 1 + 4 k ( 2 k ) ! x 2 k k = 3 ( 1 ) k + 1 ( k 1 ) ( 2 2 k 1 4 k ) ( 2 k ) ! x 2 k = k = 0 k 2 2 k + 5 + 4 k + 12 ( 2 k + 6 ) ! ( x 2 ) k k = 0 ( k + 2 ) ( 2 2 k + 5 4 k 12 ) ( 2 k + 6 ) ! ( x 2 ) k .
We now consider the sequence
S ( k ) = k 2 2 k + 5 + 4 k + 12 ( 2 k + 6 ) ! ( k + 2 ) ( 2 2 k + 5 4 k 12 ) ( 2 k + 6 ) ! = k 2 2 k + 5 + 4 k + 12 ( k + 2 ) ( 2 2 k + 5 4 k 12 ) , k 0 .
Because the inequality
S ( k ) = k 2 2 k + 5 + 4 k + 12 ( k + 2 ) ( 2 2 k + 5 4 k 12 ) < ( k + 1 ) 2 2 k + 7 + 4 k + 16 ( k + 3 ) ( 2 2 k + 7 4 k 16 ) = S ( k + 1 )
for k 0 can be reformulated as
2 2 k + 6 3 k 3 + 20 k 2 + 50 k + 52 2 2 k + 3 + k 2 + 7 k + 12 > 0
and the inequality
2 2 k + 6 > 3 k 3 + 20 k 2 + 50 k + 52 , k 0
can be verified by induction, we derive that the inequality S ( k ) < S ( k + 1 ) is true for k 0 .
Finally, employing Lemma 2 and increasing property of the sequence S ( k ) in k 0 , we acquire that the function
k = 0 k 2 2 k + 5 + 4 k + 12 ( 2 k + 6 ) ! t k k = 0 ( k + 2 ) ( 2 2 k + 5 4 k 12 ) ( 2 k + 6 ) ! t k = 2 sinh t 2 t 3 sinh t + t cosh t 2 t sinh t + t sinh 2 t 2 t cosh t cosh 2 t + 1
is increasing in t > 0 .

5. Conclusions

In this paper, we expanded the function Q ( x ) defined by (3) into the Maclaurin power series (6) around x = 0 in terms of specific determinants E 2 n and, with the aid of two monotonicity rules, proved the decreasing property of the function R ( x ) defined by (4).

Author Contributions

Writing—original draft, X.-L.L., H.-X.L. and F.Q. All authors contributed equally to the manuscript. All authors have read and agreed to the published version of the manuscript.

Funding

The second author was partially supported by the National Natural Science Foundation of China (Grant No. 62262019), by the Hainan Provincial Natural Science Foundation of China (Grant No. 823RC488), and by Haikou Science and Technology Plan Project of China (Grant No. 2022-016).

Data Availability Statement

The study did not report any data.

Acknowledgments

The authors thank anonymous referees for their careful corrections to and valuable comments on the original version of this paper.

Conflicts of Interest

The authors declare no conflict of interest.

References

  1. Gradshteyn, I.S.; Ryzhik, I.M. Table of Integrals, Series, and Products; Translated from the Russian, Eighth edition, Revised from the Seventh Edition; Zwillinger, D.; Moll, V., Translators; Elsevier/Academic Press: Amsterdam, The Netherlands, 2015. [Google Scholar] [CrossRef]
  2. Temme, N.M. Special Functions: An Introduction to Classical Functions of Mathematical Physics; A Wiley-Interscience Publication; John Wiley & Sons, Inc.: New York, NY, USA, 1996. [Google Scholar] [CrossRef]
  3. Chen, X.-Y.; Wu, L.; Lim, D.; Qi, F. Two identities and closed-form formulas for the Bernoulli numbers in terms of central factorial numbers of the second kind. Demonstr. Math. 2022, 55, 822–830. [Google Scholar] [CrossRef]
  4. Bourbaki, N. Elements of Mathematics: Functions of a Real Variable: Elementary Theory; Translated from the 1976 French Original by Philip Spain. Elements of Mathematics (Berlin); Springer: Berlin/Heidelberg, Germany, 2004. [Google Scholar] [CrossRef]
  5. Qi, F. Determinantal expressions and recursive relations of Delannoy polynomials and generalized Fibonacci polynomials. J. Nonlinear Convex Anal. 2021, 22, 1225–1239. [Google Scholar]
  6. Biernacki, M.; Krzyż, J. On the monotonity of certain functionals in the theory of analytic functions. Ann. Univ. Mariae Curie-Skłodowska Sect. A 1955, 9, 135–147. [Google Scholar]
  7. Anderson, G.D.; Vamanamurthy, M.K.; Vuorinen, M. Conformal Invariants, Inequalities, and Quasiconformal Maps; John Wiley & Sons: New York, NY, USA, 1997. [Google Scholar]
  8. Qi, F.; Taylor, P. Several series expansions for real powers and several formulas for partial Bell polynomials of sinc and sinhc functions in terms of central factorial and Stirling numbers of second kind. arXiv 2022. [Google Scholar] [CrossRef]
Disclaimer/Publisher’s Note: The statements, opinions and data contained in all publications are solely those of the individual author(s) and contributor(s) and not of MDPI and/or the editor(s). MDPI and/or the editor(s) disclaim responsibility for any injury to people or property resulting from any ideas, methods, instructions or products referred to in the content.

Share and Cite

MDPI and ACS Style

Liu, X.-L.; Long, H.-X.; Qi, F. A Series Expansion of a Logarithmic Expression and a Decreasing Property of the Ratio of Two Logarithmic Expressions Containing Sine. Mathematics 2023, 11, 3107. https://doi.org/10.3390/math11143107

AMA Style

Liu X-L, Long H-X, Qi F. A Series Expansion of a Logarithmic Expression and a Decreasing Property of the Ratio of Two Logarithmic Expressions Containing Sine. Mathematics. 2023; 11(14):3107. https://doi.org/10.3390/math11143107

Chicago/Turabian Style

Liu, Xin-Le, Hai-Xia Long, and Feng Qi. 2023. "A Series Expansion of a Logarithmic Expression and a Decreasing Property of the Ratio of Two Logarithmic Expressions Containing Sine" Mathematics 11, no. 14: 3107. https://doi.org/10.3390/math11143107

Note that from the first issue of 2016, this journal uses article numbers instead of page numbers. See further details here.

Article Metrics

Back to TopTop